LSAT and Law School Admissions Forum

Get expert LSAT preparation and law school admissions advice from PowerScore Test Preparation.

 Administrator
PowerScore Staff
  • PowerScore Staff
  • Posts: 8916
  • Joined: Feb 02, 2011
|
#38079
Complete Question Explanation
(The complete setup for this game can be found here: lsat/viewtopic.php?t=14958)

The correct answer choice is (E)

The best way to approach this question is by the process of elimination: any statement that could be true should be ruled out.

Answer choice (A) is incorrect, because F could be second (as long as K is first).

Answer choice (B) is incorrect, because G could be third. Note that if G is third, the game has only one possible solution: F L G K J H.

Answer choice (C) is incorrect, because H could be fourth (with L—fifth and G—sixth).

Answer choice (D) is incorrect, because K could be fourth. Note that if K is fourth, the game has only one possible solution: F L G K J H.

Answer choice (E): This is the correct answer choice. Given that K must be released earlier than both J and H, it is clear that K cannot be the fifth out of six released films.

Get the most out of your LSAT Prep Plus subscription.

Analyze and track your performance with our Testing and Analytics Package.